A schoolteacher would like to know whether or not toothpaste brands are used differentially in her classroom (in other words, is one brand preferred over the others?). She asks her students to report which of three brands they use: Crest, Colgate, or Aquafresh. Below are the numbers of students who use each type of toothpaste (notice there are 45 students total in her class). Test her hypothesis using an alpha level of .05.
Crest Colgate Aquafresh
24 13 8
a. What test is appropriate for this analysis?
b. State the null hypothesis:
c. State the alternative hypothesis:
d. Find the critical value:
e. Calculate the test statistic:
f. Make a decision:

Answers

Answer 1

Answer:

Step-by-step explanation:

a. What test is appropriate for this analysis?

A Chi-square  test of independence is appropriate for this analysis because it is used to compare two variables or testing relationship on categorical variables.  

b. State the null hypothesis:

The null hypothesis is the default hypothesis

[tex]\mathtt{H_o:}[/tex] There is no particular preference for any brand of toothpaste among students.

c. State the alternative hypothesis:

The alternative hypothesis is the research hypothesis which comes in place to challenge the validity of the null hypothesis.

[tex]\mathtt{H_a:}[/tex] There is particular preference for brands of toothpaste among students.

d. Find the critical value:

degree of freedom = n-1

degree of freedom = 3 - 1

degree of freedom = 2

At the level of significance ∝ = 0.05

The confidence interval = 0.95 and degree of freedom = 2, the critical value from the chi-square distribution table = 5.991

e. Calculate the test statistic:

Using the chi square test statistics; we have the following:

Crest             Colgate            Aquafresh            Total

24                        13                     8                      45

Since we have three brands. Then, for each brand, the expected value

= Total /3

= 45/3

=15

Thus:

Chi -square [tex]\mathtt{X^2 = \dfrac{(observed \ value - expected \ value)^2}{expected \ value}}[/tex]

[tex]\mathtt{X^2 = \dfrac{(24 - 15)^2}{15} + \dfrac{(13 - 15)^2}{15} + \dfrac{(8 - 15)^2}{15} }[/tex]

[tex]\mathtt{X^2 = \dfrac{81}{15} + \dfrac{4}{15} + \dfrac{49}{15} }[/tex]

[tex]\mathtt{X^2 = \dfrac{81+4+49}{15}}[/tex]

[tex]\mathtt{X^2 = \dfrac{134}{15}}[/tex]

[tex]\mathtt{X^2 =8.93 }[/tex]

f. Make a decision:

Since the chi-square value is greater than the critical value , we reject the null hypothesis and conclude that the students have  particular preference for brands of toothpaste.


Related Questions

When she graduates college, Linda will owe $43,000 in student loans. The interest rate on the federal loans is 4.5% and the rate on the private bank loans is 2%. The total interest she owes for one year was $1,585. What is the amount of each loan?

Answers

Answer:

federal loans = $29,000

private loans = $14,000

Step-by-step explanation:

x + y = 43000

.045x + .02y = 1585

x = 29,000

y = 14,000

Answer:

Amount of loan from federal : $ 29,000

Amount of loan from private bank : $ 14,000

Step-by-step explanation:

We know that Linda owes $43,000 in student loans. It is also given that the interest rate on the federal loans is 4.5%, while the interest rate on private loans is 2%, the total interest for a year being $1,585.

If Linda were to say own x dollars in federal loans, and y dollars in private loans, we know that she owns a total of $43,000, so -

x + y = 43,000

At the same time the loan interest amount is $1,585, while the interest rate on the federal loans is 4.5%, and the interest rate on private loans is 2%. The loans from each account will add to $1,585 -

0.045x + 0.02y = 1585

Let's solve the following system for x and y, the amount of each loan,

[tex]\begin{bmatrix}x+y=43000\\ 0.045x+0.02y=1585\end{bmatrix}[/tex] ( Substitute x = 43000 - y )

[tex]0.045\left(43000-y\right)+0.02y=1585[/tex] ( Simplify )

[tex]1935-0.025y=1585[/tex],

[tex]1935000-25y=1585000[/tex],

[tex]-25y=-350000[/tex],

[tex]y=14000[/tex],

[tex]x=29000[/tex]

Thus, the amount of loan from federal is $ 29,000 and the amount of loan from private bank is $ 14,000.

The manufacturer of a granola bar spends $1.20 to make each bar and sells them for $2. The manufacturer also has fixed costs each month of $8,000.

Answers

Answer:

C(x)=1.2x+8,000.

Step-by-step explanation:

C(x)=cost per unit⋅x+fixed costs.

The manufacturer has fixed costs of $8000 no matter how many drinks it produces. In addition to the fixed costs, the manufacturer also spends $1.20 to produce each drink. If we substitute these values into the general cost function, we find that the cost function when x drinks are manufactured is given by

In order to make the profits, the manufacturer must make the quantity of greater than 10000 bars.

What is a mathematical function, equation and expression?  function : In mathematics, a function from a set X to a set Y assigns to each element of X exactly one element of Y. The set X is called the domain of the function and the set Y is called the codomain of the function.expression : A mathematical expression is made up of terms (constants and variables) separated by mathematical operators.equation : A mathematical equation is used to equate two expressions.

Given is that the manufacturer of a granola bar spends $1.20 to make each bar and sells them for $2.

Suppose that you have to sell [x] number of bars to make profits. So, we can write -

{2x} - {1.20x} > {8000}

0.8x > 8000

8x > 80000

x > 10000

Therefore, in order to make the profits, the manufacturer must make the quantity of greater than 10000 bars.

To solve more questions on functions, expressions and polynomials, visit the link below -

brainly.com/question/17421223

#SPJ2

5.39 jings =15.4 hings
4.9 hings = 2.8 gings

According to the conversion rates above, how
many jings equal 1 ging?
E. 7/40
F. 5/8
G. 49/80
H. 20/7

Answers

Step-by-step explanation:

It is given that,

5.39 jings =15.4 hings  ....(1)

4.9 hings = 2.8 gings ...(2)

From equation (2), the value of 1 ging is :

[tex]1\ \text{ging} = \dfrac{4.9}{2.8}\ \text{hing}\ .....(3)[/tex]

From equation (1), the value of 1 jing is :

[tex]1\ \text{jing} = \dfrac{15.4}{5.39}\ \text{hing}\ .....(4)[/tex]

From equation (3) and (4), we get :

[tex]\dfrac{\text{1 ging}}{\text{1 jing}}=\dfrac{4.9}{2.8}\times \dfrac{5.39}{15.4}\\\\\dfrac{\text{1 ging}}{\text{1 jing}}=\dfrac{49}{80} \\\\1\ \text{ging}=\dfrac{49}{80}\ \text{ jings}[/tex]

Hence, the correct option is (g) "49/80"

If f(x)=ax+b/x and f(1)=1 and f(2)=5, what is the value of A and B?

Answers

Answer:

[tex]\huge\boxed{a=9 ; b = -8}[/tex]

Step-by-step explanation:

[tex]f(x) = \frac{ax+b}{x}[/tex]

Putting x = 1

=> [tex]f(1) = \frac{a(1)+b}{1}[/tex]

Given that f(1) = 1

=> [tex]1 = a + b[/tex]

=> [tex]a+b = 1[/tex]  -------------------(1)

Now,

Putting x = 2

=> [tex]f(2) = \frac{a(2)+b}{2}[/tex]

Given that f(2) = 5

=> [tex]5 = \frac{2a+b}{2}[/tex]

=> [tex]2a+b = 5*2[/tex]

=> [tex]2a+b = 10[/tex]  ----------------(2)

Subtracting (2) from (1)

[tex]a+b-(2a+b) = 1-10\\a+b-2a-b = -9\\a-2a = -9\\-a = -9\\a = 9[/tex]

For b , Put a = 9 in equation (1)

[tex]9+b = 1\\Subtracting \ both \ sides \ by \ 9\\b = 1-9\\b = -8[/tex]

14. Twice the sum of a number and eight

Answers

Answer: 2(x + 8) is the expression.

Use distributive property to simplify,

2x+16

I didn't know which answer you wanted so....

Answer:

2(x + 8)

Step-by-step explanation:

Hello!

Twice the sum means we multiply by 2

2

the sum of a number and eight is x + 8

2 * x + 8

Since we have to twice the sum we put x + 8 in parenthesis to show to do that first

2(x + 8)

Hope this Helps!

how many pounds are in 2 tons 1,760 ounces

Answers

Answer:

4110

Step-by-step explanation:

One ton is equal to 2000 pounds and one ounce is equal to 0.0625 pounds.

2 tons*2000 lbs per ton  = 4000 lbs

1760 ounces*0.0625 lebs per ounce = 110 lbs

4000+110=4110 lbs

A study collects samples of water from the tap in Vacaville and from bottled water available from the Nugget stores and samples their pH levels. The results are in the table below. I find a bottle marked #13 but cannot read the label for the type of water. He measures the pH and gets 6.32. What type of water do you think it is?

Answers

Answer:

see below

Step-by-step explanation:

the observed ph is 6.32

the mean pH of Tap water is shown below

                       sum of observations

Mean (Tap) = -----------------------------------

                       number of observations

 (7.24 +7.05 +7.07 +6.6 +7.28 +7.29 +7.05 +6.7 +7.16 +7.07 +7.12 +6.56

= ---------------------------------------------------------------------------------------------------------

                                                      12

= 7.016

then mean pH of bottle water is shown below

                            sum of observations

Mean (Bottles) = -----------------------------------

                             number of observations

 (5.35 + 5.29 + 5.46 + 5.4 + 5.95 + 6.22 + 5.43 +5.48 +6.06 +5.33 +5.46 +5.41)

= --------------------------------------------------------------------------------------------------------------

                                                      12

= 5.57

theoretically.. the higher the pH values should be between 0 to 14.

based from the above results, the mean tap water has an average of 7.016 and by looking at the pH chart... its a neutral or pure water.

while the average pH Bottles has 5.57, this means its more acidic water, or by looking at the pH chart its an acid rain water.

a 6.32pH is below pure water, based on the chart looks like a urine/saliva.


What is the volume of a cube with a side length of
of a unit?

Answers

It’s d times it three times length width height

Solve for y.
-1 = 8+3y
Simplify you answer as much as possible.

Answers

Answer:

-3

Step-by-step explanation:

[tex]8+3y = -1\\3y = -9\\y = -3[/tex]

Answer:

y = -3

Step-by-step explanation:

-1=3y+8

3y+8=-1

3y=-9

y=-3

What is 5 feet and 11 inches in inches

Answers

Answer:

60

Step-by-step explanation:

5 is 60 inch

h(x) = x2 + 1 k(x) = x – 2

Evaluate 3h(2) + 2k(3) =

Answers

Answer:

17

Step-by-step explanation:

[tex]h(x) =x^2 +1\\k(x)=x-2\\\\3h(2)+2k(3)\\\\h(2)= ?\\k(3)=?\\\\h(2) = (2)^2 +1\\= 4+1\\h(2)=5\\\\\\k(3)= 3-2\\k(3) = 1\\\\3h(2) +2k(3)\\\\= 3(5)+2(1)\\=15+2\\3h(2)+2k(3) = 17[/tex]

5, 9, and 17

Step-by-step explanation:

Transform the polar equation to a Cartesian (rectangular) equation: r= 4sinθ

options include:

x^2+y^2 = 4y

x^2+y^2 = -4

x^2+y^2 = 4

x^2+y^2 = -4y

Answers

Answer:

  x^2 +y^2 = 4y

Step-by-step explanation:

Using the usual translation relations, we have ...

  r^2 = x^2+y^2

  x = r·cos(θ)

  y = r·sin(θ)

Substituting for sin(θ) the equation becomes ...

  r = 4sin(θ)

  r = 4(y/r)

  r^2 = 4y

Then, substituting for r^2 we get ...

  x^2 +y^2 = 4y . . . . . matches the first choice

write a thirdthird-degree polynomial expression that has only two terms with a leading term that has a coefficient of five and a constant of negative two ​

Answers

Answer:

5x^3-2

[tex]ax^{3} +bx^{2} +cx+d\\5x^{3}-given\\ d=-2-given\\5x^{3} -2[/tex]

Answer: [tex]5x^3 - 2[/tex]

Explanation:

The two terms are [tex]5x^3[/tex] and [tex]2[/tex]. Terms are separated by either a plus or minus.

We can write it as [tex]5x^3+(-2)[/tex] which is an equivalent form. Here the two terms are [tex]5x^3[/tex] and [tex]-2[/tex]. This is because adding a negative is the same as subtracting.

The coefficient is the number to the left of the variable.

The degree is the largest exponent, which helps form the leading term.

The third degree polynomial written above is considered a cubic binomial. "Cubic"  refers to the third degree, while "binomial" means there are 2 terms.

We can write something like [tex]5x^3[/tex] as 5x^3 when it comes to computer settings.

Jilk Inc.'s contribution margin ratio is 62% and its fixed monthly expenses are $45,000. Assuming that the fixed monthly expenses do not change, what is the best estimate of the company's net operating income in a month when sales are $132,000?

Answers

Answer: $ 36,840.

Step-by-step explanation:

contribution margin=62% =0.62

fixed monthly expenses = $45,000

Sales =  $132,000

We assume that the fixed monthly expenses do not change.

Then, company's net operating income = (contribution margin×Sales )-fixed monthly expenses

=$( (0.62×132000)-45000 )

= $ (81840-45000)

= $ 36,840

Hence, the best estimate of the company's net operating income in a month when sales are $132,000 is $ 36,840.

The formula for the area of a square is s2, where s is the side length of the square. What is the area of a square with a side length of 6 centimeters? Do not include units in your answer.

Answers

Answer:

36

Step-by-step explanation:

formula of area for square:

A=s^2

s=6

A=6^2

A=36

Answer:

36

Step-by-step explanation:

I got it right

what are the steps required to determine the equation of a quadratic function given its zeros and a point?​

Answers

Answer:

Below

Step-by-step explanation:

The quadratic equations form is:

● ax^2+bx+c

Using the zeroes, we can write a factored form.

● a (x-x') (x-x")

x and x' are the zeroes

■■■■■■■■■■■■■■■■■■■■■■■■■■

●y = a (x-x') (x-x")

x' and x" are khown but a is not.

We are given a point so replace x and y with its coordinates to find a.

So the steps are:

● 1) Write the factored form of the quadratic equation

● 2) replace x' and x" with their values.

● 3) replace x and y with the coordinates of a khwon point.

● 4) solve the equation for a.

The steps are write the factored form of the quadratic equation then, replace x' and x" with their values. To replace x and y with the coordinates of a known point. To solve the equation for a.

What is a quadratic equation?

A quadratic equation is the second-order degree algebraic expression in a variable. the standard form of this expression is  ax² + bx + c = 0 where a. b are coefficients and x is the variable and c is a constant.

Using the zeroes, we can write a factored form;

a (x-x') (x-x")

x and x' are the zeroes

y = a (x-x') (x-x")

x' and x" are known but a is not.

We are given a point so replace x and y with their coordinates to find a.

So the steps are:

1) Write the factored form of the quadratic equation

2) To replace x' and x" with their values.

3) To replace x and y with the coordinates of a known point.

4) To solve the equation for a.

Learn more about quadratic equations;

brainly.com/question/13197897

#SPJ2

A machine fills boxes weighing Y lb with X lb of salt, where X and Y are normal with mean 100 lb and 5 lb and standard deviation 1 lb and 0.5 lb, respectively. What percent of filled boxes weighing between 104 lb and 106 lb are to be expected?
a. 67%
b. None
c. 37%
d. 57%

Answers

Answer:

Option b. None is the correct option.

The Answer is 63%

Step-by-step explanation:

To solve for this question, we would be using the z score formula

The formula for calculating a z-score is given as:

z = (x-μ)/σ,

where

x is the raw score

μ is the population mean

σ is the population standard deviation.

We have boxes X and Y. So we will be combining both boxes

Mean of X = 100 lb

Mean of Y = 5 lb

Total mean = 100 + 5 = 105lb

Standard deviation for X = 1 lb

Standard deviation for Y = 0.5 lb

Remember Variance = Standard deviation ²

Variance for X = 1lb² = 1

Variance for Y = 0.5² = 0.25

Total variance = 1 + 0.25 = 1.25

Total standard deviation = √Total variance

= √1.25

Solving our question, we were asked to find the percent of filled boxes weighing between 104 lb and 106 lb are to be expected. Hence,

For 104lb

z = (x-μ)/σ,

z = 104 - 105 / √25

z = -0.89443

Using z score table ,

P( x = z)

P ( x = 104) = P( z = -0.89443) = 0.18555

For 1061b

z = (x-μ)/σ,

z = 106 - 105 / √25

z = 0.89443

Using z score table ,

P( x = z)

P ( x = 106) = P( z = 0.89443) = 0.81445

P(104 ≤ Z ≤ 106) = 0.81445 - 0.18555

= 0.6289

Converting to percentage, we have :

0.6289 × 100 = 62.89%

Approximately = 63 %

Therefore, the percent of filled boxes weighing between 104 lb and 106 lb that are to be expected is 63%

Since there is no 63% in the option, the correct answer is Option b. None.

The percent of filled boxes weighing between 104 lb and 106 lb is to be expected will be 63%.

What is a normal distribution?

It is also called the Gaussian Distribution. It is the most important continuous probability distribution. The curve looks like a bell, so it is also called a bell curve.

The z-score is a numerical measurement used in statistics of the value's relationship to the mean of a group of values, measured in terms of standards from the mean.

A machine fills boxes weighing Y lb with X lb of salt, where X and Y are normal with a mean of 100 lb and 5 lb and standard deviation of 1 lb and 0.5 lb, respectively.

The percent of filled boxes weighing between 104 lb and 106 lb is to be expected will be

Then the Variance will be

[tex]Var = \sigma ^2[/tex]

Then for X, we have

[tex]Var (X) = 1^2 = 1[/tex]

Then for Y, we have

[tex]Var (Y) = 0.5^2 = 0.25[/tex]

Then the total variance will be

[tex]Total \ Var (X+Y) = 1 + 0.25 = 1.25[/tex]

The total standard deviation will be

[tex]\sigma _T = \sqrt{Var(X+Y)}\\\\\sigma _T = \sqrt{1.25}[/tex]

For 104 lb, then

[tex]z = \dfrac{104-105}{\sqrt{25}} = -0.89443\\\\P(x = 104) = 0.18555[/tex]

For 106 lb, then

[tex]z = \dfrac{106-105}{\sqrt{25}} = 0.89443\\\\P(x = 106) = 0.81445[/tex]

Then

[tex]P(104 \leq Z \leq 106) = 0.81445 - 0.18555 = 0.6289 \ or \ 62.89\%[/tex]

Approximately, 63%.

More about the normal distribution link is given below.

https://brainly.com/question/12421652

A particle moves according to a law of motion s = f(t), t ≥ 0, where t is measured in seconds and s in feet. (If an answer does not exist, enter DNE.) f(t) = t3 − 8t2 + 27t

Answers

The question is not clear, but it is possible to obtain distance, s, from the given function. This, I would show.

Answer:

s = 17 units

Step-by-step explanation:

Given f(t) = t³ - 8t² + 27t

Differentiating f(t), we have

f'(t) = 3t² - 16 t + 27

At t = 0

f'(t) = 27

This is the required obtainaible distance, s.

(a^8)3/2 in simplest form ​

Answers

Answer:

[tex]\large\boxed{\frac{3}{2}a^{8}}[/tex]

Step-by-step explanation:

([tex]a^{8}[/tex]) * [tex]\frac{3}{2}[/tex]

Remove the parenthesis by multiplying

[tex]\frac{3}{2}[/tex][tex]a^{8}[/tex]

This expression cannot be simplified further

[tex]\large\boxed{\frac{3}{2}a^{8}}[/tex]

Hope this helps :)

a company should stop making a part internally and buy externally when

Answers

Answer:

Make-or-Buy Decision

Step-by-step explanation:

What is the error in this problem

Answers

Answer:

12). LM = 37.1 units

13). c = 4.6 mi

Step-by-step explanation:

12). LM² = 23² + 20² - 2(23)(20)cos(119)°

    LM² = 529 + 400 - 920cos(119)°

    LM² = 929 - 920cos(119)°

    LM = [tex]\sqrt{929+446.03}[/tex]

          = [tex]\sqrt{1375.03}[/tex]

          = 37.08

          ≈ 37.1 units

13). c² = 5.4² + 3.6² - 2(5.4)(3.6)cos(58)°

    c² = 29.16 + 12.96 - 38.88cos(58)°

    c² = 42.12 - 38.88cos(58)°

    c = [tex]\sqrt{42.12-20.603}[/tex]

    c = [tex]\sqrt{21.517}[/tex]

    c = 4.6386

    c ≈ 4.6 mi

If f(x) = 2x2 – 3x – 1, then f(-1)=

Answers

ANSWER:
Given:f(x)=2x^2-3x-1
Then,f(-1)=2(-1)^2-3(-1)-1
f(-1)=2(1)+3-1
f(-1)=5-1
f(-1)=4


HOPE IT HELPS!!!!!!
PLEASE MARK BRAINLIEST!!!!!

The value of function at x= -1 is f(-1) = 4.

We have the function as

f(x) = 2x² - 3x -1

To find the value of f(-1) when f(x) = 2x² - 3x -1, we substitute x = -1 into the expression:

f(-1) = 2(-1)² - 3(-1) - 1

      = 2(1) + 3 - 1

      = 2 + 3 - 1

      = 4.

Therefore, the value of function at x= -1 is f(-1) = 4.

Learn more about Function here:

https://brainly.com/question/32020999

#SPJ6

Find the sum of (5x3 + 3x2 - 5x + 4) and (8x3 -5x2 + 8x + 9)

Answers

Answer:

(5x³+3x²-5x+4) + (8x³-5x²+8x+9)

= 5x³+3x²-5x+4 +8x³-5x²+8x+9

= 5x³+8x³+3x²-5x²-5x+8x+4+9

= 13x³-2x²+3x+13

Hope this helps

if u have question let me know in comments ^_^

F
19) The points (6,5), (7,2), (9,6), and (10,3) are vertices of an inscribed square.
A)(x - 8)2-(y - 4)2 = 5
B) (x – 8)2 + (y - 4)2 = 15
C) (X + 8)2 + (y + 4)2 = 5
D) (x - 8)2 + (y - 4)2 = 5
Find an equation for the circle

Answers

Answer:

The equation of circle is [tex](x-8)^2+(y-4)^2=5[/tex]

(D) is correct option.

Step-by-step explanation:

Given that,

Points (6,5), (7,2), (9,6) and (10,3) are vertices of an inscribed square.

We need to calculate the distance between (7,2) and (9,6)

Using formula of distance

[tex]d=\sqrt{(x_{2}-x_{1})^2+(y_{2}-y_{1})^2}[/tex]

Put the value into the formula

[tex]d^2=(9-7)^2+(6-2)^2[/tex]

[tex]d^2=20\ m[/tex]

The radius will be

[tex]r^2=\dfrac{20}{4}[/tex]

[tex]r^2=5[/tex]

We need to calculate the center of the point (7,2) and (9,6)

Using formula of center point

For x axis,

[tex]h=\dfrac{x_{2}+x_{1}}{2}[/tex]

Put the value into the formula

[tex]h=\dfrac{9+7}{2}[/tex]

[tex]h=\dfrac{16}{2}[/tex]

[tex]h=8[/tex]

For y axis,

[tex]k=\dfrac{y_{2}+y_{1}}{2}[/tex]

Put the value into the formula

[tex]k=\dfrac{6+2}{2}[/tex]

[tex]k=\dfrac{8}{2}[/tex]

[tex]k=4[/tex]

We need to find the equation for the circle

Using formula of equation of circle

[tex](x-h)^2+(y-k)^2=r^2[/tex]

Put the value into the formula

[tex](x-8)^2+(y-4)^2=5[/tex]

Hence, The equation of circle is [tex](x-8)^2+(y-4)^2=5[/tex]

(D) is correct option.

Which of the following is NOT a property of a paralleogram? * The opposite sides are equal. The opposite angles are equal. Each diagonal bisects the parallogram. The diagonals of all parallograms bisect each other at 90 degree angles. I will give brainliest

Answers

Answer:

The diagonals of all parallelograms do not bisect each other at 90 degree angles.

Step-by-step explanation:

Which of the following is the solution to the inequality below? -5x — 10 -6 B. x > -2 C. x <-6 D. x < -2

Answers

Answer:

x > -6

Step-by-step explanation:

-5x — 10 < 20

Add 10 to each side

-5x — 10+10 < 20+10

-5x < 30

Divide each side by -5, remembering to flip the inequality

-5x/-5 > 30/-5

x > -6

Answer:

x>-6

Step-by-step explanation:

[tex]-5x - 10 < 20\\\\\mathrm{Add\:}10\mathrm{\:to\:both\:sides}\\\\-5x-10+10<20+10\\\\\mathrm{Multiply\:both\:sides\:by\:-1\:\left(reverse\:the\:inequality\right)}\\\\\left(-5x\right)\left(-1\right)>30\left(-1\right)\\\\\mathrm{Simplify}\\\\5x>-30\\\\\mathrm{Divide\:both\:sides\:by\:}5\\\\\frac{5x}{5}>\frac{-30}{5}\\\\x>-6[/tex]

Find the number of pieces of floor tiles each measuring 26cm long and 10cm wide needed to lay a floor measuring 260m long and 15m wide

Answers

Answer:

150,000

Step-by-step explanation:

1 m = 100 cm

260 m = 260 * 100 cm = 26000 cm

15 m = 15 * 100 cm = 1500 cm

area of floor = LW = 26000 cm * 1500 cm = 39,000,000 cm^2

area of 1 tile = 26 cm + 10 cm = 260 cm^2

number of tiles needed = 39,000,000/260 = 150,000

Answer: 150,000 tiles

Correct answer is 150000 tiles. Hope this helps ya

According to a government study among adults in the 25- to 34-year age group, the mean amount spent per year on reading and entertainment is $1,999. Assume that the distribution of the amounts spent follows the normal distribution with a standard deviation of $574. (Round your z-score computation to 2 decimal places and final answers to 2 decimal places.) What percent of the adults spend more than $2,550 per year on reading and entertainment?

Answers

Answer:

The probability is  [tex]P(X > x ) = 0.19215[/tex]

Step-by-step explanation:

From the question we are told that

   Th The population mean [tex]\mu = \$ 1,999[/tex]

    The  standard deviation is  [tex]\sigma = \$ 574[/tex]

    The  values considered is  [tex]x = \$ 2,500[/tex]

Given that the distribution of the amounts spent follows the normal distribution then the  percent of the adults spend more than $2,550 per year on reading and entertainment is mathematically represented as

    [tex]P(X > x ) = P(\frac{ X - \mu}{\sigma } > \frac{ x - \mu}{\sigma } )[/tex]

Generally  

            [tex]X - \mu}{\sigma } = Z (The \ standardized \ value \ of \ X )[/tex]

So

      [tex]P(X > x ) = P(Z > \frac{ x - \mu}{\sigma } )[/tex]

substituting values

      [tex]P(X > 2500 ) = P(Z > \frac{ 2500 - 1999}{574 } )[/tex]

      [tex]P(X > 2500 ) = P(Z >0.87 )[/tex]

From the normal distribution table the value of [tex]P(Z >0.87 )[/tex] is  

       [tex]P(Z >0.87 ) = 0.19215[/tex]

Thus  

       [tex]P(X > x ) = 0.19215[/tex]

A train is running at the speed of 90 mph. The length of the train is 300 ft. How long would it take to cross a railway platform 492 ft long?

Answers

Answer:

Time = 1.45152 seconds

Step-by-step explanation:

1 foot = 0.000189 mile

300 ft = 300*0.000189

300 ft = 0.0567 miles

492 ft = 492*0.000189

492 ft = 0.092988 miles

Distance left to be covered by the train

= 0.092988-0.0567

= 0.036288 miles

Speed= 90mph

Time taken = distance/speed

Time taken= 0.036288/90

Time = 4.032*10^-4 hour

Time = 4.032*10^-4*60*60

Time = 1.45152 seconds

if (ax+b)(x-3) = 4x^2+cx-9 for all values of x, what is the value of c? a) -9 b) -6 c) 6 d) 9

Answers

Answer:

c=-9

Step-by-step explanation:

Hello,

[tex](ax+b)(x-3)=ax(x-3)+b(x-3)=ax^2-3ax+bx-3b\\\\=ax^2+(b-3a)x+(-3b) \\\\\text{And it should be equal to } 4x^2+cx-9[/tex]

We can identify the like terms so:

a = 4

b-3a = c

3b = 9 <=> b = 3

So c = 3 - 3*4 = 3-12 = -9

Hope this helps.

Do not hesitate if you need further explanation.

Thank you

Other Questions
Jack is 10 years old.Kylie is 17 years old.Vanessa is 23 years old.Kylie and Vanessa share 16 in the ratio of their ages.Kylie gives 20% of her share to Jack.Vanessa gives a quarter of her share to Jack.How much money does Jack receive?plz answer me step by step plzz plz Which expression is equivalent to 73 75? 72 77 1 over 7 to the 2nd power 1 over 7 to the 7th power Which of the following is a salt that could be generated by combining a weak acid and a weak base? Select the correct answer below: a) NaCl b) Na2SO4 c) NH4NO3 d) NH4F Allison bought jelly beans to share with her friends. She bought pounds of blueberry jelly beans and pounds of lemon jelly beans. If she gave pounds of jelly beans away to her friends, how many pounds of jelly beans does Allison have left? The two common properties of all solids are fixed _____ and _____. How do attributes affect function? The temperature at midnight is shown. The outside temperature decreases 2.3 C over the next two hours. What is the outside temperature at 2 A.M. ? Question 8(Multiple Choice Worth 1 points)(06.05 MC)A paper cup is dropped and its landing position is recorded. The cup can land on the side, on the open end, or on the closed end. The results of 20 trials are shown in the table below:Paper Cup Experiment# of times occurredOpenHT IIIClosedSideHT IIIBased on the table, which of the following best compares the experimental probability of the cup landing on its open end with the experimental probability of the cup landing on its closed end?The probabilities are equal.The probability of landing on the open end is greater.The probability of landing on the closed end is greater.O No conclusion can be made. Dr. Potter provides vaccinations against polio and measles. Each polio vaccination consists of 6 doses, and each measles vaccination consists of 3 doses. Last year, Dr. Potter gave a total of 60 vaccinations that consisted of a total of 225 doses. How many more measles vaccines did Mr. Potter give than polio? Show All Work !! Variable versus absorption costing Colorado Business Tools, manufactures calculators. Costs incurred in making 9,500 calculators in February included 29,450 of fixed manufacturing overhead. The total absorption cost per calculator was $10.25. Required: a. Calculate the variable cost per calculator. b. The ending inventory of pocket calculators was 750 units higher at the end of the month than at the beginning of the month. By how much and in what direction (higher or lower) would operating income for the month of February be different under variable costing than under absorption costing? c. Express the pocket calculator cost in a cost formula. PLEASE ANSWER !! WILL GIVE BRAINLIEST! Consider the exponential functions f, g, and h, defined as shown. Place the three functions in order from the fastest decreasing average rate of change to the slowest decreasing average rate of change on the interval [0, 3]. All of the following statements about the maps above are TRUE except: A. These maps illustrate census data. B. In 2000, many states had more people under 18 than they had in 1990. C. In 2000, many states had fewer people under 18 than they had in 1990. D. These maps represent the number of people under 18 living in the US in 1990 and 2000. Hello people, please if you can give me a Hint with this, l only get half of the marks, what i am doing wrong here? thanks which of the following best describes the bases of a cylinder? A. Congruent B. Polygons C. Parallel D. Discs (Check All That Apply) What did modern democracies learn from the Romans?how to limit a ruler's power by establishing a constitutional monarchyto maintain a lasting government, allow only the rich and powerful to make and enforce laws for all the citizenshow to safeguard against tyranny by dividing the government's power among different partshow to protect the rights of citizens by allowing the leaders to have absolute power What is the area of a regular hexagon with an apothem 16.5 inches long and a side 19 inches long? Round the answer to the nearest tenth. A. 625.3 in.2 B. 940.5 in.2 C. 156.3 in.2 D. 1,875.8 in.2 Simply. Who ever answers this will be marked Brainlist. Part of the population of 6,250 elk at a wildlife preserve is infected with a parasite. A random sample of 50 elk shows that 6 of them are infected. How many elk are likely to be infected? Based on the sample, there will likely be infected elk in the wildlife preserve. what are the next 3 terms in the sequence? 0.8,1,1.2,1.4,1.6.... How many triangles exist with the given side lengths? 2mm,6mm,10mm